The measure of ∠LMN is 158°.
To find the measure of ∠LMN, we can use the angle bisector theorem and angle addition postulate.
Let's denote the measure of ∠LMN as x.
According to the angle bisector theorem, since →MP bisects ∠LMN, we have:
m∠LMP / m∠NMP = LM / LN
Since →MP bisects ∠LMP, we know that m∠LMP = m∠NMP. Therefore, we can rewrite the equation as:
m∠LMP / m∠LMP = LM / LN
This simplifies to:
1 = LM / LN
So, we can conclude that LM = LN.
Now, let's consider ∠QMP. Since →MR bisects ∠QMP, we know that m∠RMP = m∠QMP.
Using the angle addition postulate, we can write:
m∠RMP + m∠LMP + m∠LMN = 180°
Substituting the given value, m∠RMP = 21°, we have:
21° + m∠LMP + x = 180°
Simplifying this equation, we get:
m∠LMP + x = 159°
Now, we know that m∠LMP = m∠NMP, and since LM = LN, we can substitute LM for LN:
m∠NMP + x = 159°
Since LM = LN, we can also substitute 1 for LM/LN:
1 + x = 159°
Subtracting 1 from both sides:
x = 159° - 1°
x = 158°
Therefore, the measure of ∠LMN is 158°.
To know more about measure, refer here:
https://brainly.com/question/28913275
#SPJ4
i need help please
its pretty urgent
Fatima is taking a test that is 45 minutes long.
She finishes the test at 10:50 a.m.
What time did she start the test?
Answer:10:05
Step-by-step explanation:
subtract 45 from 50!
Answer:
She started the test at 10:05 am
Step-by-step explanation:
Take the end time and subtract the time of the test
10:50-45 = 10:05
She started the test at 10:05 am
x+ 5y +3z = 4
4y - z = 3
6x - 2y + 4z = 0
Share £20 in the ratio of 3:2
Answer:
12 and 8
Step-by-step explanation:
3/8 (3/4+5/6) - 1/2
Answer:
3/32
Step-by-step explanation:
1. 3/8 (3/4 + 5/6) - 1/2
2. 3/8 x 19/12 - 1/2
3. 19/32 - 1/2
4. 3/32
~~~~~~~~~~~~~~~~~~~~~~~~~~~~~~~~~~~~~~~~~~~~~~~~~~~~~~~~~~~~~~~~~~~
First, you do the operation order. In this equation, parentheses goes first. You add the fractions to get 19/12.
Then, we multiply the sum from the parentheses and 3/8, 19/32.
Finally, we subtract the product we got from the parentheses and 3/8 with 1/2, each lead us to our final answer 3/32.
~~~Hope this helps~~~
The Key Club sells hot chocolate and coffee at the varsity soccer games. In one game, they sold $200 worth of hot drinks. They need to report how many of each type of drink was sold for their club records. Nancy knows that they used 295 cups that evening. If hot chocolate sells for $.75 and coffee sells for $.50, how many more cups of hot chocolate were sold than coffee? Write your answer as a complete sentence.
3. (3 pts) Find the general solution of the following homogeneous differential equations. 2xyy' + (x² - y²) = 0
The general solution of the given homogeneous differential equation is:y = ±x√(Cx² + 2)
The given differential equation is: 2xyy' + (x² - y²) = 0
We have to find the general solution of the given homogeneous differential equation. To solve the above differential equation, we will use the substitution method.
Put y = vx and y' = v + xv'
Differentiating both sides w.r.t x: y' = v + xv' ⇒ v' = (y' - v)/x
Differentiating again w.r.t x: y'' = v' + xv'' + v' ⇒ y'' = (y'' - 2y'v + v² + xv')/x
Substituting these values in the given differential equation:
2x(v)(v + xv') + (x² - v²x²) = 0
2v + 2x²v' + x - v² = 0
2x²v' + 2v/x - v³/x = -1/x
We can write the above differential equation as:
2x²dv/dx + 2v/x = v³/x - 1/x
Separating the variables and integrating both sides:
∫dx/x = ∫[v/(v² - 2)]dv/x
⇒ ln |x| + ln |v² - 2| + C
⇒ ln |x(v² - 2)| = ln |Cx|
⇒ v² - 2 = Cx² (where C is a constant)
⇒ v = ±√(Cx² + 2)
Putting the value of v in y = vx, we get:
y = x√(Cx² + 2) and y = -x√(Cx² + 2)
To know more about differential equation visit:
https://brainly.com/question/25731911
#SPJ11
PLEASE HELP!!!
Find two consecutive positive integers whose product is 156. Let x represent the first number.
Write a quadratic equation to model the verbal sentence. Identify the integers.
12 and 13 are the the two consecutive positive integers whose product is 156.
Let x and x+1 are the two consecutive positive integers whose product is 156.
Let x represent the first number and x+1 is second number
x(x+1) = 156
Expanding the left side:
x² + x = 156
Bringing everything to one side:
x² + x - 156 = 0
The equation is in the form of quadratic
x² + 13 x - 12x - 156 = 0
x(x+13)+12(x+13)=0
(x-12)(x+13)=0
x=12 and x=-13
Therefore, the two consecutive positive integers are 12 and 13, since their product is 156.
To learn more on Quadratic equation click:
https://brainly.com/question/17177510
#SPJ1
PLEASE ANSWER THIS ASAP! What is the length of a diagonal of a cube with a side length of 3 cm?
Answer:
√27
Step-by-step explanation:
3×√3 = 5.196
√27 = 5.196
Answer:
27
Step-by-step explanation:
StartRoot 27 EndRoot cm
help please ineed this
Answer:
(3,1)
Step-by-step explanation:
Tameca already has $55
dollars in her savings account. If
she puts $5 per week in her
account, write and solve an inequality
inequality to find out how man
weeks she must save to have a
least $100 in her account.
Answer:
5x+55=100
Step-by-step explanation:
Prove that the angle that an arc of a circle ubtend at the Centre i twice that which it ubtend at any point on the remaining part of the circumference
Therefore , the solution of the given problem of circumference statement is proved by the following below explanation.
What is circumference?Any great circle's circumference, or the distance between the center of the sphere and any plane running through it, is measured in meters. Any large circle that passes through a pole-designated location is referred to as a meridian.
Here,
An arc PQ of a circle that subtends angles POQ at its center O and PAQ at one of its points A on the circumference.
To prove : <POQ = 2<PAQ
To prove this theorem we consider the arc AB in three different situations, minor arc AB, major arc AB and semi-circle AB.
Construction:
Join the line AO extended to B.
Proof :
<BOQ = ZOAQ + ZAQO .....(1)
Also, in A OAQ,
OA = OQ
Therefore,
[Radii of a circle]
ZOAQ = ZOQA
[Angles opposite to equal sides are equal]
<BOQ = 2ZOAQ
.......(2)
Similarly, BOP = 220AP
.(3)
Adding 2 & 3, we get,
<BOP + <BOQ = 2(ZOAP + ZOAQ)
<POQ = 2<PAQ
.......(4)
For the case 3, where PQ is the major arc, equation 4 is replaced by
Reflex angle, <POQ = 2ZPAQ
Therefore , the solution of the given problem of circumference statement is proved by the following below explanation.
To know more about circumference , visit
https://brainly.com/question/28757341
#SPJ4
a. (5) The demand function for a good X is Qx= m-3Px+2Py, where m is income, Px is the price of X, Py is the price of a related good Y and Qx is the demand for X. Income and prices are all positive. X
The demand function for good X is Qx = m - 3Px + 2Py, where Qx is the quantity demanded of X, m is income, Px is the price of X, and Py is the price of a related good Y. The equation shows that the demand for X is inversely related to its price and directly related to the price of Y. Income, price of X, and price of Y collectively affect the overall demand for X.
The demand function for good X is given by Qx = m - 3Px + 2Py, where Qx represents the quantity demanded of good X, m is the income, Px is the price of good X, and Py is the price of a related good Y. In this equation, the income and prices are assumed to be positive.
To determine the demand for good X, we can analyze the equation. The coefficient -3 in front of Px indicates that the demand for good X is inversely related to its price. As the price of X increases, the quantity demanded of X decreases, assuming other factors remain constant. On the other hand, the coefficient 2 in front of Py indicates that the demand for good X is directly related to the price of the related good Y. If the price of Y increases, the quantity demanded of X also increases, assuming other factors remain constant.
Furthermore, the term (m - 3Px + 2Py) represents the overall effect of income, price of X, and price of Y on the quantity demanded of X. If income (m) increases, the quantity demanded of X increases. If the price of X (Px) increases, the quantity demanded of X decreases. If the price of Y (Py) increases, the quantity demanded of X increases.
To know more about the factors affecting the demand for good X, refer here:
https://brainly.com/question/32566433#
#SPJ11
If C and D are square matrices of size m X m, then which of the following statement is not true? (1) det(C.D) = det(C). det (D), (2) det(D. Dt) = (det(D))², (3) det(AdjD) = (detD)^m-1 (4) det(5C + 4D) = 5 det(C) + 4det (D) 4 (5) det(C-¹)= 1/det (c)
A) (4) B) (3) C) (2) D) (5) E) (1)
If C and D are square matrices of size m x m, then the statement that is not true is \((5) det(C^-1) = 1/det(C)\). The determinant is a mathematical object that is used in linear algebra to define scalar invariants of square matrices.
The determinant can also be viewed as the matrix's scaling factor, which changes the magnitude of a matrix's linear transformation in order to describe how the linear transformation alters the volume of any shape. The adjugate matrix is the transpose of the matrix of cofactors for a square matrix. The product of two matrices A and B is a matrix C such that A is a matrix with m rows and n columns and B is a matrix with n rows and p columns. A matrix with the same number of rows and columns is called a square matrix. It is a matrix in which the number of rows and columns is the same, i.e., it has an equal number of rows and columns. The determinant of a 2x2 matrix can be found using the formula ad - bc. If C and D are square matrices of size m X m, then the statement that is not true is \((5) det(C^{-1}) = 1/det(C)\).
To learn more about matrices, visit:
https://brainly.com/question/29024761
#SPJ11
prove that x = 0 is the only solution to ax = 0 if and only if the rank of the p ×q matrix a equals q, the number of unknowns.
The statement that x = 0 is the only solution to ax = 0 if and only if the rank of the p × q matrix a equals q, the number of unknowns, can be proven as follows:
First, assume that x = 0 is the only solution to ax = 0. This means that there are no nonzero solutions to the equation, implying that the null space of the matrix a is trivial (only containing the zero vector). By the rank-nullity theorem, the nullity of a matrix is equal to the dimension of its null space. Since the null space of a is trivial, its nullity is zero.
Next, consider the rank of the matrix a. By the rank-nullity theorem, the rank of a plus its nullity is equal to the number of columns in a, which is q. Therefore, if the nullity is zero, the rank of a must be equal to q.
Conversely, if the rank of the matrix a equals q, it implies that the nullity of a is zero. This means that the only solution to the equation ax = 0 is the zero vector, x = 0.
x = 0 is the only solution to ax = 0 if and only if the rank of the matrix a equals q, the number of unknowns. This result highlights the relationship between the null space and the rank of a matrix, providing insights into the solutions of linear equations and the properties of the matrix.
know more about rank of the matrix :brainly.com/question/30748258
#SPJ11
Ryan is writing a composition for homework. He decides to keep track of the number of
sentences he writes compared to the time minutes he works. The graph below shows the data he collected.
At What Rate Does Ryan Write His Composition?
Answer:
Your answer will be 1.5
Step-by-step explanation:
I used my calculator to find out, your welcome:)
A rectangular restaurant kitchen has an area of 80 square meters and a perimeter of 36 meters. What are the dimensions of the kitchen?
Answer:
Step-by-step explanation
Frist, the area = ab = 30 m^2 and the perimeter = 2(a + b) = 34 m or a + b = 17 m (2). Solving (1) and (2), a = 15 m and b = 2 m. Since it is a rectangle, the dimensions are (all in m) so the answer is: 15, 2, 15, 2.
Answer:
THe kitchen is 8 by 10
Step-by-step explanation:
x = width
y = length
Area = xy = 80 m²
Perimeter = 2x + 2y = 36 m
2x = 36 - 2y
x = 18 - y substitute into equation 1
(18 - y)(y) = 80
-y² + 18y - 80 = 0 find roots of y by factoring
y² - 18y + 80 = 0
(y - 8)(y - 10) = 0
y = 8, 10
Since xy = 80, then:
x = 80/10 = 8, or, x = 80/8 = 10
Now you have your dimensions: 8 and 10
To check the answers:
8 x 10 = 80 m²
2(8) + 2 (10) = 36 m
Answers are correct!
Find the slope of the line going through the points (1,3) and (4,9) The slope of the lines is m = (For example, slope -2)
Please help due soon :)
Draw a possible net for the figure above.
Please need this done like rn
What is the exact value of tan(pi/4)?
1
The exact value of tan(π4) tan ( π 4 ) is 1 .
You spend $40 on 5 pounds of concrete. What is the unit rate in dollars per pound?
a
$8 per pound
b
$5 per pound
c
$35 per pound
d
$0.125 per pound
Answer:
$8 per pound
Step-by-step explanation:
40 ÷ 5 = 8
$40 per 5 pound= 8
$8 per 1 pound = 8
HELP!!!!!Your boss offers you a job with two pay options:#1: Earn $100 per week plus 7.5% commission on sales.Option #2: Earn $350 a week, with no commission.If you sold $145,000 worth of merchandise during the month of February, what would be the difference in the two salary options for the month?
The difference in salary options for the month of February would be $9,875.
In Option #1, you would earn $100 per week plus 7.5% commission on sales. In Option #2, you would earn a fixed salary of $350 per week, with no commission. To calculate the difference in salary options for the month of February, we need to calculate the earnings for each option and then subtract Option #2 from Option #1.
In Option #1, the commission on sales would be 7.5% of $145,000, which is $10,875. The weekly earnings would be $100 per week. Since there are four weeks in February, the total earnings for Option #1 would be $100 * 4 + $10,875 = $11,275.
In Option #2, the weekly earnings would be a fixed $350 per week. So, the total earnings for Option #2 in February would be $350 * 4 = $1,400.
To find the difference in salary options, we subtract the earnings of Option #2 from Option #1: $11,275 - $1,400 = $9,875.
Therefore, the difference in salary options for the month of February would be $9,875.
To learn more about difference click here: brainly.com/question/31059956
#SPJ11
Solve for X
5(x + 8) = -10
Answer:
x = -10
Step-by-step explanation:
5 (x + 8) = -10
5x + 40 = -10
5x = -50
x= -10
Answer:
x= -10
Step-by-step explanation:
5(x+8)=-10
5x+40=-10
move the 40 to the other side
5x=-10-40
5x= -50
divide 5 by both sides to get rid of the x.
x=-10
Example 6: The area of a rectangle is 3.6 meters. Th
base of the rectangle is 3 meters. Which equation
can be used to find h, the height of the rectangle?
PLEASE HELP
A. 3.6 + h = 3
B. 3.6 = 3h
B. 3.6 = h - 3
D.3.6 = 4
Answer:
B. 3.6 = 3h
Step-by-step explanation:
Area of rectangle formula :-
A = L × W
3.6 = 3 × W OR the answer choic it gave is 3.6 = 3h.
Hope this helps, thank you !!
What is the highest possible number of positive real zero/s does the polynomial function f(x) = x4 – 3x3 – 4x2 + 6x - 1 have?
a. 3
b. 1
c. 2
d. 4
Answer: The answer to the question is $\boxed{\text{(d)}\ 4}$. This is because a polynomial of degree 4 can have at most 4 zeros, and those zeros can be either real or complex numbers. Since the question asks for the maximum number of positive real zeros, the answer is $\boxed{\text{(d)}\ 4}$.
Step-by-step explanation: The maximum number of positive real zeros that a polynomial function of degree 4 can have is 4. This is because a polynomial of degree 4 can have at most 4 zeros, and those zeros can be either real or complex numbers. Since the question asks for the maximum number of positive real zeros, the answer is $\boxed{\text{(d)}\ 4}$.
A landscaping company charges $110 for 4 hours of lawn care. They charge the same amount of money for every hour.
Which table represents the relationship between hours of lawn care and the amount of money the company charges?
Responses
Hours of lawn care Amount charged (dollars)
8 $220
10 $275
12 $330
14 $385Hours of lawn care Amount charged (dollars) 8 $220 10 $275 12 $330 14 $385 ,
Hours of lawn care Amount charged (dollars)
2 $110
4 $165
6 $220
8 $275
Hours of lawn care Amount charged (dollars) 2 $110 4 $165 6 $220 8 $275
Hours of lawn care Amount charged (dollars)
4 $110
5 $115
6 $121
7 $128
Hours of lawn care Amount charged (dollars) 4 $110 5 $115 6 $121 7 $128
Hours of lawn care Amount charged (dollars)
3 $110
4 $110
5 $110
6 $110
Hours of lawn care Amount charged (dollars) 3 $110 4 $110 5 $110 6 $110
The table representing the relationship between hours of lawn care and the amount of money the company charges is:
8 $220
10 $275
12 $330
14 $385.
Define Equations.Equations are mathematical expressions with two algebraic expressions flanking the equals (=) symbol on either side. It demonstrates the equality of the relationship between the expressions printed on the left and right sides. LHS = RHS (left hand side equals right hand side) is the most widely accepted theorem.
Let x be the charge of lawn care for 1 hour.
Given, charge for 4 hours of lawn care =$110
So, x= Amount charged/ Hours of lawn care
x=110/4
x=27.5
Now, the company charges $27.5 for 1 hour of lawn care.
So now we can find the amount charged by the company for
"y" no. of hours by using the equation. x × y = z
Here x, y and z are variables, where x is the charge of lawn care for 1 hour. y is the number of hours of lawn care and z is the total amount
charged by the company.
To know more about equations visit:
https://brainly.com/question/29657983
#SPJ1
What is the slope of (1, -3/4) and (4, -3)
Answer:
1-5
Step-by-step explanation:
Which point lies on the graph of the function shown below y=-x^2+5x-3
A.(1,4)
B.(0,3)
C.(2,3)
D.(-2,2)
Answer:
C: (2,3)
Step-by-step explanation:
to solve this algebraically just plug in the x and y values of the points and see if they are true, but here is the graph to juse show you
A limousine service has 15 cars available for rides: 8 are Lincoln Town Cars and the remainder are Cadillac models. Suppose an event organizer requests five limousines from this company. How many different collections C of five limousines are possible?
If A limousine service has 15 cars available for rides: then the number of different collections C of 5 limousines are possible is 3003 .
To find the number of different collections C of five limousines, we can use combinations. A combination is a way to choose a subset of items from a larger set, without regard to order.
Let the number of Cadillac models n.
Then, 15 - 8 = 7 cars are Cadillacs. So, n = 7.
The number of different collections of 5 limousines from the 15 cars is given by the number of combinations of 5 items taken from a set of 15 items:
⇒ ¹⁵C₅ = 15!/(5!×(15 - 5)!) = 15!/(5!×10!) = 3003.
Therefore , there are 3003 different collections C of five limousines that the event organizer can choose from the limousine service's fleet of 15 cars.
Learn more about Combination here
https://brainly.com/question/30362022
#SPJ4